rank the following types of radiation in order of increasing energy per photon: a. radar signals b. radiation in a microwave oven c. gamma rays from nuclear radiation

Answers

Answer 1

The order of increasing energy per photon is radar signals (lowest), radiation in a microwave oven, and gamma rays from nuclear radiation (highest).

To rank the following types of radiation in order of increasing energy per photon, we have: a. radar signals, b. radiation in a microwave oven, c. gamma rays from nuclear radiation.

1. Radar signals: These have the lowest energy per photon among the three mentioned types of radiation. Radar signals are a type of radio wave, which are on the lower end of the electromagnetic spectrum.

2. Radiation in a microwave oven: Microwaves have higher energy per photon compared to radar signals but lower than gamma rays. They are located between radio waves and infrared waves on the electromagnetic spectrum.

3. Gamma rays from nuclear radiation: These have the highest energy per photon among the three types of radiation. Gamma rays are on the higher end of the electromagnetic spectrum and are produced by nuclear reactions, cosmic rays, and other high-energy processes.

Learn more about energy:

https://brainly.com/question/13881533

#SPJ11


Related Questions

a 650 nm laser shines through a diffraction grating. the first bright band is 0.54 m from the center. another laser is only deflected to 0.42 m from the center. what is the wavelength of this light?

Answers

The second laser has a wavelength of around 835.71 nm.

What is the diffraction grating's level formula?

N = 1/ d, where d is the grating spacing, is the number of slits per metre on the grating. At a given order and wavelength, the angle of diffraction rises as d value falls. In other words, as the number of slits per metre grows, so does the angle of diffraction.

d sinθ = mλ

sinθ₁ = (0.54 m) / d

For the second laser, m = 1 again and the distance from the center is 0.42 m. We can solve for sinθ₂:

sinθ₂ = (0.42 m) / d

Since the spacing of the diffraction grating is the same for both lasers, we can set the two equations equal to each other and solve for λ:

d sinθ₁ = d sinθ₂

(0.54 m) / λ = (0.42 m) / λ

Simplifying, we get:

λ = (0.54 m * 650 nm) / 0.42 m

λ = 835.71 nm

To know more about wavelength visit:-

https://brainly.com/question/13533093

#SPJ1

Question:

A laser with a wavelength of 650 nm shines through a diffraction grating. The first bright band is observed at a distance of 0.54 m from the center. Another laser is shone through the same grating and is deflected to a distance of 0.42 m from the center. What is the wavelength of the second laser?

PLEASE ANSWER ASAP
1. How many atoms are present in 8.500 mole of chlorine atoms?
2. Determine the mass (g) of 15.50 mole of oxygen.
3. Determine the number of moles of helium in 1.953 x 108 g of helium.
4. Calculate the number of atoms in 147.82 g of sulfur.
5. Determine the molar mass of Co.
6. Determine the formula mass of Ca3(PO4)2.
IT WOULD BE HELPFUL

Answers

The number of atoms in 147.82 g of sulphur is 2.772 x 10²⁴. In 1.953 x 10⁸ g of helium, there are 4.883 × 10⁷ moles of helium. 15.50 moles of oxygen weigh 248 g. Calcium phosphate's formula mass would be 310.18 g/mol.

How many atoms make up 1 gramme?

The quantity of atoms or molecules per gramme of atomic weight is known as Avogadro's number, which is 6.022 × 10²³/mole. One mole of hydrogen comprises 6.022 × 10²³ hydrogen atoms for one gramme of hydrogen with an atomic weight of one gramme.

The Avogadro's number states that 1 mole of any substance contains 6.022 x 10²³ particles. Therefore, 8.500 moles of chlorine atoms would contain:

8.500 moles * 6.022 x 10²³ atoms/mole = 5.1167 x 10²⁴ atoms of chlorine.

The molar mass of oxygen is 16.00 g/mol. Therefore, 15.50 moles of oxygen would have a mass of:

15.50 moles * 16.00 g/mol = 248 g

So the mass of 15.50 mole of oxygen is 248 g.

The molar mass of helium is 4.00 g/mol. Therefore, 1.953 x 10⁸ g of helium would contain:

1.953 x 10⁸ g / 4.00 g/mol = 4.883 x 10⁷ moles of helium.

The molar mass of sulfur is 32.06 g/mol. Therefore, 147.82 g of sulfur would contain:

147.82 g / 32.06 g/mol = 4.6055 moles of sulfur. Therefore, the formula mass of Calcium phosphate would be:

(340.08 g/mol) + (230.97 g/mol) + (8*16.00 g/mol) = 310.18 g/mol.

To know more about formula visit:-

https://brainly.com/question/29792091

#SPJ1

when a high voltage is applied to a low-pressure gas, causing it to glow, it will emit what type of spectrum? a. li

Answers

When a high voltage is applied to a low-pressure gas and it starts to glow, it will emit an emission line spectrum.

This spectrum consists of bright, narrow lines at specific wavelengths, which are characteristic of the element or molecules in the gas. This is due to the electrons in the gas being excited to higher energy levels and then falling back down to lower energy levels, emitting photons of light at specific wavelengths corresponding to the energy differences between the levels. The resulting emission spectrum can be used to identify the elements or molecules present in the gas.

Learn more about spectrum

https://brainly.com/question/6836691

#SPJ4

what happens to each bulb if the switch is closed? match the words in the left column to the appropriate blanks in the sentences on the right. resethelp once the switch is closed, the current flows blankbecau

Answers

When the switch is closed, the circuit is completed, and the current starts flowing. The behavior of each bulb depends on the arrangement of the bulbs and the switch in the circuit.

If the bulbs are arranged in a series circuit, the current flows through both bulbs in the same direction. In this case, the voltage across each bulb is proportional to its resistance. Therefore, if the bulbs have the same resistance, they will have the same voltage across them. If one bulb has a higher resistance than the other, it will have a higher voltage across it. The current flowing through both bulbs will be the same, but the voltage across them will differ.

If the bulbs are arranged in a parallel circuit, the current splits into different branches and each branch contains a bulb. In this case, the voltage across each bulb is the same, and the current flowing through each bulb is proportional to its resistance. Therefore, if one bulb has a higher resistance than the other, it will have a lower current flowing through it. If one bulb has a lower resistance than the other, it will have a higher current flowing through it. The voltage across both bulbs stays the same, and no other bulb becomes short-circuited.

In conclusion, the behavior of each bulb depends on the arrangement of the circuit. If the bulbs are arranged in a series circuit, the voltage across them differs, and the current flowing through them is the same. If the bulbs are arranged in a parallel circuit, the voltage across them is the same, and the current flowing through them differs.

To learn more about circuit

https://brainly.com/question/27206933

#SPJ4

Complete question:

What happens to each bulb if the switch is closed? Match the words in the left column to the appropriate blanks in the sentences on the right. Res through both bulbs Once the switch is closed, the current flows because only through bulb A only through bulb B the voltage across it becomes zero the voltages across them stay the same another bulb becomes short-circuited no branch of a circuit is opened.

the loudness of sound, measured in decibels (db), is calculated using the formula , where l is the loudness, and i is the intensity of the sound.what is the intensity of a fire alarm that measures 125db loud? round your answer to the nearest hundredth.intensity

Answers

The intensity of the fire alarm that measures 125 dB loud is approximately 3.16 W/[tex]m^{2}[/tex].


To calculate the intensity (I) of a fire alarm that measures 125 dB loud, we need to use the formula for loudness (L):

L = 10 * log10(I / Io)

In this formula, L is the loudness (in dB), I is the intensity of the sound, and Io is the reference intensity ([tex]10^{-12}[/tex] W/[tex]m^{2}[/tex]). We are given L = 125 dB and we want to find I. First, we need to rearrange the formula to solve for I:

I = Io *[tex]10^{L/10}[/tex]

Now, plug in the given values:

I = 10^-12 *[tex]10^{125/10}[/tex]
I = 10^-12 * [tex]10^{12.5}[/tex]
I ≈ 3.16 W/[tex]m^{2}[/tex]

The intensity of the fire alarm that measures 125 dB loud is approximately 3.16 W/[tex]m^{2}[/tex]

Know more about   intensity   here:

https://brainly.com/question/28145811

#SPJ11

1. which angular velocity was non-zero and what was the sign? explain how this makes sense given the right-hand rule for the angular velocity.

Answers

Clockwise angular velocity was  non-zero and had a positive sign. So, the correct answer is D.

The right-hand rule for angular velocity asserts that if the right hand's thumb is pointing in the direction of the axis of rotation, then the direction of the angular velocity vector is given by the direction in which the right hand's fingers curl.

This makes sense in this situation. As a result, the angular velocity vector will point in the same direction as the rotation's axis, and it will be positive when the angular velocity is positive.

In physics, engineering, and other sciences, the right-hand rule for angular velocity is a helpful tool for visualising the direction of the angular velocity vector.

This rule allows us to quickly ascertain the direction and sign of the angular velocity in any given situation.

Complete Question:

Which angular velocity was non-zero and what was the sign? Explain how this makes sense given the right-hand rule for the angular velocity.

A.  Counterclockwise, Positive

B.  Clockwise, Negative

C. Counterclockwise, Negative

D. Clockwise, Positive

To learn more about velocity visit:

https://brainly.com/question/80295

#SPJ4

T or F: If one cuts a current carrying wire, the flow of electricity will spill out into the air

Answers

False because when a current-carrying wire is cut, the circuit is broken and the flow of electricity is interrupted. The electrons in the wire will stop moving, and there will be no flow of electricity in the air.

The current in the wire is carried by electrons, which are negatively charged particles that are tightly bound to the wire. When the wire is cut, the electrons can no longer flow in a continuous path and the current will stop. However, there may be a brief spark or arc if the wire is cut while there is still a high voltage present, as the electrons try to jump across the gap in the wire. But once the voltage dissipates, the current flow will stop completely.

To learn more about electricity please visit:

https://brainly.com/question/12791045

#SPJ11

False. Cutting a wire that carries current won't cause electricity to discharge into the atmosphere. But the circuit will be broken, and no longer will power be flowing.

A wire produces a magnetic field as current runs through it. The electrons are kept flowing by this magnetic field in a certain direction, and when the wire is severed, the circuit is broken and the electrons cease to move. Nevertheless, if the wire is cut in a way that sparks or if the wire is improperly insulated, the energy may arc or leap to conductive material nearby, potentially posing a threat. Care must be used when handling wires that carry current, and proper safety precautions must be taken.

learn more about electricity here:

https://brainly.com/question/15800304  

#SPJ11

a 1 meter long solenoid with 200 turns carries 2a of current . calculate the magnetic field on axis.

Answers

The magnetic field on the axis of the solenoid is 5.03 × 10⁻⁴ T.

The magnetic field on the axis of a solenoid can be calculated using the formula:

B = μ₀ * n * I

Where B denotes the intensity of the magnetic field, 0 denotes the permeability of empty space, n denotes the number of turns per unit length, and I is the current flowing through the solenoid.

In this case, the solenoid is 1 meter long and has 200 turns, so n = 200 turns / 1 meter = 200 turns/meter. The solenoid is delivering 2A of current.

The value of μ₀ is a constant, equal to 4π × 10⁻⁷ T·m/A

When we enter these values into the formula, we get:

B = μ₀ * n * I

= 4π × 10⁻⁷ T·m/A * 200 turns/m * 2A

= 5.03 × 10⁻⁴ T

Therefore, the magnetic field on the axis of the solenoid is 5.03 × 10⁻⁴ T.

Learn more about  magnetic field

https://brainly.com/question/14848188

#SPJ4

magnetic field on the axis of the solenoid is approximately 0.005 T

Solution -  Hi! To calculate the magnetic field on the axis of a solenoid, you can use the formula:

Magnetic field (B) = μ₀ * n * I . (applicable for ideal long solenoid)

where μ₀ is the permeability of free space (approximately 4π x 10^-7 Tm/A), n is the number of turns per unit length, and I is the current.

In your case, the solenoid is 1 meter long with 200 turns and carries a 2 A current. To find n, divide the number of turns by the length:

n = 200 turns / 1 m = 200 turns/m

Now, plug the values into the formula:

B = (4π x 10^-7 Tm/A) * (200 turns/m) * (2 A)

B ≈ 0.005 T

The magnetic field on the axis of the solenoid is approximately 0.005 T (Tesla).

To learn more about solenoid, click on this

brainly.com/question/22043434

#SPJ11

A particle beam is made up of many protons, each with a kinetic energy of 3. 25times 10-15 J. A proton has a mass of 1. 673 times 10-27 kg and a charge of +1. 602 times 10-19 C. What is the magnitude of a uniform electric field that will stop these protons in a distance of 2 m?

Answers

The magnitude of the uniform electric field required to stop the protons in a distance of 2 m is 1.10 x 10^32 N/C.

To solve this problem, we need to use the equation for the work done by an electric field on a charged particle:

W = qEd

First, we need to calculate the velocity of the protons:

[tex]K = 1/2 mv^2 \\v = sqrt(2K/m)[/tex]

Plugging in the values, we get:

[tex]v = sqrt(2 * 3.25 * 10^{-15} J / 1.673 * 10^{-27} kg)\\v = 5.94 * 10^6 m/s[/tex]

Time it takes for the proton to stop:

[tex]t = d/v \\t = 2 m / 5.94 * 10^6 m/s \\t = 3.37 * 10^-7 s[/tex]

Finally, we can use the time and the acceleration due to the electric field to calculate the electric field strength:

[tex]a = v/t \\a = 5.94 * 10^6 m/s / 3.37 * 10^{-7} s\\a = 1.76 * 10^13 m/s^2[/tex]

[tex]E = a/q \\E = 1.76 * 10^{13} m/s^2 / 1.602 * 10^{-19} C\\E = 1.10 * 10^{32} N/C[/tex]

Therefore, the magnitude of the uniform electric field required to stop the protons in a distance of 2 m is 1.10 x 10^32 N/C.

To know more about electric field strength, here

brainly.com/question/28227168

#SPJ4

a laser beam takes 45.5 ms to be reflected back from a totally reflecting sail on a spacecraft. how far away is the sail?

Answers

The sail is approximately 6,832,500 meters away from the laser source.

To determine the distance between the laser source and the totally reflecting sail on a spacecraft, we'll use the time it takes for the laser beam to be reflected back, which is 45.5 ms (milliseconds).

Since the laser beam travels to the sail and back, we must account for the round trip. The speed of light is approximately 3.0 x 10^8 meters per second (m/s).

First, convert 45.5 ms to seconds: 45.5 ms × (1 s / 1000 ms) = 0.0455 s.

Next, calculate the total distance the laser beam travels during this time: distance = speed × time, so distance = (3.0 x 10^8 m/s) × 0.0455 s ≈ 13,665,000 meters.

Finally, divide the total distance by 2 to find the distance between the laser source and the sail: 13,665,000 meters / 2 ≈ 6,832,500 meters.

To learn more about : meters

https://brainly.com/question/24445340

#SPJ11

a proton moving in the plane of the page has a kinetic energy of 6.00 mev. a magnetic field of 1.00 t is directed into the page. the proton enters the magnetic field with its velocity vector at an angle?

Answers

The velocity of a proton when it enters the magnetic field is [tex]1.58 × 10^7 m/s.[/tex]

What is the velocity vector at an angle?

We can use the equation for the magnetic force on a charged particle to solve this problem:

F = qvBsinθ

where F is the magnetic force, q is the charge of the particle, v is its velocity, B is the magnetic field, and θ is the angle between the velocity vector and the magnetic field.

Since the proton has a positive charge, it will experience a force perpendicular to its velocity vector, which will cause it to move in a circular path in the plane of the page.

The centripetal force required to keep the proton in a circular path is provided by the magnetic force, so we can equate the two forces:

[tex]F = mv^2/r[/tex]

where m is the mass of the proton, and r is the radius of the circular path.

Equating these two forces, we get:

[tex]qvBsinθ = mv^2/r[/tex]

Solving for the radius, we get:

[tex]r = mv/qBsinθ[/tex]

Substituting the given values, we get:

[tex]r = (1.67 × 10^-27 kg)(3 × 10^8 m/s)/((1.6 × 10^-19 C)(1.00 T)sinθ) = 3.32 × 10^-3/sinθ meters[/tex]

The kinetic energy of the proton is also given, which can be related to its speed v:

[tex]K = (1/2)mv^2[/tex]

[tex]v = sqrt(2K/m) = sqrt((2)(6.00 × 10^6 eV)(1.6 × 10^-19 J/eV)/(1.67 × 10^-27 kg)) = 1.58 × 10^7 m/s[/tex]

Substituting this value for v, we get:

[tex]r = (1.67 × 10^-27 kg)(1.58 × 10^7 m/s)/((1.6 × 10^-19 C)(1.00 T)sinθ) = 1.05 × 10^-3/sinθ meters[/tex]

Finally, we can solve for sinθ:

[tex]sinθ = r/(1.05 × 10^-3 meters) = (3.32 × 10^-3 meters)/(1.05 × 10^-3 meters) = 3.15[/tex]

However, since sinθ can only range from -1 to 1, this value is not physically meaningful. Therefore, we can conclude that the proton cannot enter the magnetic field at any angle that will result in a circular path.

Learn more about magnetic field

brainly.com/question/14848188

#SPJ11

the magnetic force per meter on a wire is measured to be only 55% of its maximum possible value. what is the angle between the wire and the magnetic field?

Answers

The angle between the wire and the magnetic field is approximately 33.6 degrees.

To find the angle between the wire and the magnetic field, we will use the following formula for the magnetic force per meter on a wire:

F = BIL sin(θ)

where F is the magnetic force per meter, B is the magnetic field strength, I is the current flowing through the wire, L is the length of the wire, and θ is the angle between the wire and the magnetic field.

Given that the magnetic force is only 55% of its maximum possible value, we can write the equation as:

0.55 * F_max = BIL sin(θ)

The maximum force occurs when sin(θ) = 1, which means:

F_max = BIL

Now, we can substitute F_max back into our first equation:

0.55 * BIL = BIL sin(θ)

Now, divide both sides by BIL:

0.55 = sin(θ)

Finally, to find the angle θ, take the inverse sine (sin^(-1)) of both sides:

θ = sin^(-1)(0.55)

θ ≈ 33.6 degrees

So approximately 33.6 degrees is the angle between the wire and the magnetic field.

More on magnetic field: https://brainly.com/question/15567206

#SPJ11

A car with a mass of 1000 kg is traveling east at 4 m/s. Another car with a mass of 500 kg is traveling west at a speed of 3 m/s. The two cars collide. After the collision, the 1000 kg car has a velocity of 1 m/s east. What is the velocity of the 500 kg car after the collision?

Answers

Answer: the velocity of the 500 kg car after the collision is 3 m/s to the east.

Explanation:

Initial momentum = (mass of car 1 x velocity of car 1) + (mass of car 2 x velocity of car 2)

Initial momentum = (1000 kg x 4 m/s) + (500 kg x -3 m/s)   (Note that we use a negative velocity for car 2 because it is traveling in the opposite direction)

Initial momentum = 4000 kg m/s - 1500 kg m/s = 2500 kg m/s

After the collision, the total mass and total momentum of the system remain the same.

Final momentum = (mass of car 1 x velocity of car 1) + (mass of car 2 x velocity of car 2)

Final momentum = (1000 kg x 1 m/s) + (500 kg x v)  (where v is the velocity of the 500 kg car after the collision)

Final momentum = 1000 kg m/s + 500v

Since the total momentum is conserved, we can set the initial momentum equal to the final momentum:

Initial momentum = Final momentum

2500 kg m/s = 1000 kg m/s + 500v

Solving for v, we get:

v = (2500 kg m/s - 1000 kg m/s) / 500 kg

v = 3 m/s

??

solid forms of ice last longer because there is more weight with less surface area. (True or False)

Answers

The solid forms of ice last longer because there is more weight with less surface area. This statement is false.

Factors like temperature, shape, size, humidity and impurities are some of the factor decides the time for which the ice survives. Even though larger ice particles may have more surface area than solid forms of ice, this does not always imply that they will persist longer.

In reality, due to the insulating effect of the ice itself, larger ice formations, like glaciers, can melt more quickly. In the end, a complex combination of physical, chemical, and environmental elements determines how long ice will last.

To know more about Melting of ice, visit,

https://brainly.com/question/1079154

#SPJ4

if a wrench is 28 cm long, what force perpendicular to the wrench must the mechanic exert at its end? express your answer with the appropriate units.

Answers

If a wrench is 28 cm long, the mechanic must exert a force of 3.57 N perpendicular to the wrench at its end.

To solve this problem, we need to use the formula:

Force = Torque / Distance

where Torque is the product of force and distance. In this case, we know the distance (28 cm), but we need to find the torque first.

Assuming that the mechanic is applying a force perpendicular to the wrench, the torque can be calculated as:

Torque = Force x Distance

where Force is the force exerted by the mechanic at the end of the wrench and Distance is the length of the wrench (28 cm).

Rearranging the formula, we get:

Force = Torque / Distance

Substituting the values, we get:

Force = (Torque) / (Distance)
Force = (1 N.m) / (0.28 m)
Force = 3.57 N

Therefore, the mechanic must exert a force of 3.57 N perpendicular to the wrench at its end. The unit for force is Newtons (N).

More on force: https://brainly.com/question/22597079

#SPJ11


HELP PLEASE Light travels to Earth from space as a/an_________wave.

O Mechanical
OSound
O Electromagnetic
O Longitudinal

Answers

Answer:

electromagnetic wave.

Explanation:

You can see light from the moon, distant stars, and galaxies because light is an electromagnetic wave. Electromagnetic waves are waves that can travel through matter or through empty space.

Answer: C) Electromagnetic wave

Explanation: It can't be D) Longitudinal because there is no such thing as a longitudinal wave that has to do with space. It wouldn't be mechanical cuz a mechanical doesn't have anything to do with light, neither sound.

Thus, the answer is C) Electromagnetic

what is the wavelength of a radio photon from an am radio station that broadcasts at 1270 kilohertz? express your answer to three significant figures and include the appropriate units.

Answers

The wavelength of a radio photon from an AM radio station broadcasting at 1270 kilohertz is 236 meters.

To find the wavelength of a radio photon from an AM radio station broadcasting at 1270 kilohertz, we can use the formula:
wavelength (λ) = speed of light (c) / frequency (f)

1. First, we need to convert the frequency from kilohertz to hertz:
1270 kilohertz = 1270 * 10³ hertz = 1,270,000 hertz

2. Next, we will use the speed of light, which is approximately 3.00 * 10⁸ meters per second (m/s).

3. Now, we can plug in the values into the formula:
wavelength (λ) = (3.00 * 10⁸ m/s) / (1,270,000 Hz)

4. Calculate the wavelength:
λ ≈ 236.22 meters

5. Finally, express the answer to three significant figures and include the appropriate units:
λ ≈ 236 meters

Learn more about wavelength:

https://brainly.com/question/10750459

#SPJ11

when the distance between two charges is halved, the electrical force between the charges is reduced by 1/4. quadruples. halves. doubles. none of the above choices are correct.

Answers

When the distance between two charges is halved, the electrical force between the charges quadruples. This is due to the inverse square relationship between distance and electrical force, which means that when distance is halved, the force increases by a factor of 4.



The electrical force between the charges quadruples when the distance between them is halved. This is due to Coulomb's Law, which states that the electrical force (F) between two charges (q1 and q2) is directly proportional to the product of the charges and inversely proportional to the square of the distance (r) between them. Mathematically, it can be expressed as:

F = k * (q1 * q2) / r^2

When the distance (r) is halved, the denominator (r^2) becomes 1/4 of its original value, which causes the electrical force (F) to be 4 times greater, or quadruple.

To learn more about quadruples please visit:

https://brainly.com/question/7966538

#SPJ11

what are planetary rings made of, and how do they differ among the four jovian planets? match the terms in the left column to the appropriate blanks in the sentences on the right. resethelp planetary rings are made up of countless small particles composed of blank and blank.target 1 of 10target 2 of 10 all rings lie in the blank. rings' particles have blank orbits.target 3 of 10target 4 of 10 blank's rings are the brightest and widest among jovian planets. their particles consist most of blank.target 5 of 10target 6 of 10 blank's rings are mostly dusty and less visible.target 7 of 10 blank and blank both have narrow bright rings diveded by very sparse dusty rings in between.target 8 of 10target 9 of 10 blank's narrow rings show irregularities in form of brighter arcs, as if the rings were incomplete

Answers

Numerous tiny ice and rock fragments make up the planet's ring system. The four jovian planets differ from one another in terms of colour and shape.

All rings lie in the planet's equatorial plane. Jupiter's rings are the brightest and widest among jovian planets. Their particles consist mostly of small, dark rock fragments. Saturn's rings are mostly dusty and less visible. Uranus and Neptune both have narrow bright rings divided by very sparse dusty rings in between. Uranus's narrow rings show irregularities in the form of brighter arcs, as if the rings were incomplete.

Planetary rings are made up of countless small particles composed of ice and rock. All rings lie in the equatorial plane. Rings' particles have elliptical orbits. Saturn's rings are the brightest and widest among jovian planets. Their particles consist mostly of ice. Jupiter's rings are mostly dusty and less visible. Uranus and Neptune both have narrow bright rings divided by very sparse dusty rings in between. Neptune's narrow rings show irregularities in the form of brighter arcs, as if the rings were incomplete.

For more such questions on Planetary rings , Visit:

https://brainly.com/question/9828009

#SPJ11

if the human body has an average density of 983 kg/m3 , what fraction of a person is submerged when floating gently in fresh water?

Answers

Roughly half of a person's body is submerged when they are floating gently in fresh water.

When an object is floating in water, it displaces an amount of water equal to its weight. If the weight of the displaced water is greater than the weight of the object, the object will float, and if it is less, the object will sink.

The fraction of a person that is submerged while floating in fresh water depends on the ratio of their weight to the weight of the water displaced by their body.

Assuming that the volume of a person is constant, we can calculate the weight of a person as:

Weight = density x volume x gravity

where:

density = 983 [tex]kg/m^3[/tex] (average density of human body)

volume = the volume of a person (assumed to be constant)

gravity = 9.81 [tex]m/s^2[/tex] (acceleration due to gravity)

Let's assume the weight of a person is 70 kg, then their volume would be:

Volume = Weight / (density x gravity)

Volume = 70 / (983 x 9.81)

Volume = 0.00716 [tex]m^3[/tex]

Now, let's consider the weight of the water displaced by the person's body. The weight of the water displaced is equal to the weight of the person, which is 70 kg.

Therefore, the fraction of the person's body that is submerged in water is:

Fraction submerged = Weight of water displaced / Total weight

Fraction submerged = 70 / (70 + weight of water in the submerged part of the body)

Since the person is floating gently in water, we can assume that only a small part of their body is submerged, and we can neglect the weight of water in the submerged part of the body.

Thus, we can approximate the fraction submerged as:

Fraction submerged ≈ Weight of water displaced / Total weight

Fraction submerged ≈ 70 / (70 + 70)

Fraction submerged ≈ 0.5 or 50%

Therefore, roughly half of a person's body is submerged when they are floating gently in fresh water.

Learn more about submerged

https://brainly.com/question/30690757

#SPJ4

As a planet orbits a star, it makes a big ellipse, but its gravity has a similar effect on the star, causing the star to make a small star. this is called

Answers

As a planet orbits a star, it makes a big ellipse, but its gravity has a similar effect on the star, causing the star to make a small star. This is called the "gravitational wobble" or "stellar wobble".

As a planet orbits a star, it follows an elliptical path due to the gravitational pull of the star. The shape of the planet's orbit is determined by the balance between the gravitational force of the star and the planet's own motion. However, the planet's gravity also affects the star, causing it to move slightly in response to the planet's pull. This motion of the star is much smaller than that of the planet, but it is still measurable and can be observed. This phenomenon is known as the planet's gravitational influence on the star, which causes the star to wobble slightly. This effect is used by astronomers to detect and study exoplanets orbiting distant stars.

To learn more about gravitational wobble, refer:-

https://brainly.com/question/31168966

#SPJ11

The phenomenon that occurs when a planet orbits a star, causing both the planet and the star to make elliptical motions due to their mutual gravitational effects.

This phenomenon is known as the "wobble" or "stellar wobble" and is caused by the gravitational interaction between a planet and its star. As a planet orbits a star, it exerts a gravitational force on the star, causing it to move slightly in response. This movement results in a small, periodic shift in the star's spectral lines, which can be detected by astronomers.

By analyzing this shift, astronomers can determine the presence, size, and orbital characteristics of planets around other stars. At the same time, the planet's gravity also affects the star, causing the star to make a smaller elliptical motion in response. This mutual gravitational interaction results in the observed stellar wobble.

To know more about stellar wobble:

https://brainly.com/question/29542172

#SPJ11

how fast must a nonrelativistic electron move so its de broglie wavelength is the same as the wavelength of a 3.4-ev photon?

Answers

Answer:

1990.47 m/s

Explanation:

Answer: the answer is in the screen shots

Explanation:

the acceleration due to gravity on the moon’s surface is one-sixth that on earth. what net force would be required to accelerate a 20-kg object at 6.0 m/s2 on the moon?

Answers

To determine the net force required to accelerate a 20-kg object at 6.0 m/s² on the moon, we need to consider the acceleration due to gravity on the moon and the object's mass.

The acceleration due to gravity on the moon is one-sixth that on Earth. Since the acceleration due to gravity on Earth is approximately 9.81 m/s², the acceleration due to gravity on the moon is (1/6) * 9.81 m/s² ≈ 1.63 m/s².

Now, we can use Newton's second law of motion, F = m * a, to find the net force required for the given acceleration on the moon. Here, m = 20 kg (mass of the object) and a = 6.0 m/s² (desired acceleration).

Net force (F) = 20 kg * 6.0 m/s² = 120 N.

So, the net force required to accelerate a 20-kg object at 6.0 m/s² on the moon is 120 N.

To know more about Newton's second law of motion:

https://brainly.com/question/27712854

#SPJ11

please help answer all A through D will give 100 points

Entropy has some interesting properties. Calculate the change in entropy for the following situations. For these small temperature changes, you can use the original temperature to find the changes in entropy.
A: Heating 1.0 kg of water from 272 K to 274 K.
B:Heating 1.0 kg of water from 353 K to 354 K.
C:Heating 1.0 kg of lead from 273 K to 274 K.
D:Completely melting 1.0 kg of ice at 273 K.

Answers

The changes in entropy are: A) 30.8 J/K, B) 11.8 J/K, C) 0.47 J/K and D) 1223 J/K

What is entropy?

Entropy is a thermodynamic quantity that describes the degree of disorder or randomness in a system. It is a measure of the number of possible arrangements or microstates that a system can have, given its macroscopic properties like temperature, pressure, and volume.

The change in entropy can be calculated using the following formula:

ΔS = Q/T

Where ΔS is the change in entropy, Q is the heat absorbed or released, and T is the temperature in Kelvin.

A) Heating 1.0 kg of water from 272 K to 274 K:

The specific heat capacity of water is 4.184 J/(g·K), so the heat absorbed can be calculated as follows:

Q = m × c × ΔT

Q = 1000 g × 4.184 J/(g·K) × (274 K - 272 K)

Q = 8,368 J

The change in entropy is:

ΔS = Q/T

ΔS = 8,368 J / 272 K

ΔS = 30.8 J/K

B) Heating 1.0 kg of water from 353 K to 354 K:

Using the same formula as before:

Q = m × c × ΔT

Q = 1000 g × 4.184 J/(g·K) × (354 K - 353 K)

Q = 4,184 J

The change in entropy is:

ΔS = Q/T

ΔS = 4,184 J / 353 K

ΔS = 11.8 J/K

C) Heating 1.0 kg of lead from 273 K to 274 K:

The specific heat capacity of lead is 0.128 J/(g·K), so the heat absorbed can be calculated as follows:

Q = m × c × ΔT

Q = 1000 g × 0.128 J/(g·K) × (274 K - 273 K)

Q = 128 J

The change in entropy is:

ΔS = Q/T

ΔS = 128 J / 273 K

ΔS = 0.47 J/K

D) Completely melting 1.0 kg of ice at 273 K:

The heat of fusion of ice is 333.55 J/g, so the heat absorbed can be calculated as follows:

Q = m × ΔH

Q = 1000 g × 333.55 J/g

Q = 333,550 J

The change in entropy is:

ΔS = Q/T

ΔS = 333,550 J / 273 K

ΔS = 1223 J/K

Therefore, the changes in entropy are:

A) 30.8 J/K

B) 11.8 J/K

C) 0.47 J/K

D) 1223 J/K

To know more about entropy, visit:

https://brainly.com/question/31047254

#SPJ1

if a sound wave transitions from one medium to another, which transition would result in a shortening of the wavelength of the sound wave?

Answers

If a sound wave transitions from one medium to another, a transition from a medium with a higher speed of sound to a medium with a lower speed of sound would result in a shortening of the wavelength of the sound wave.


1. When a sound wave enters a new medium, its frequency remains constant.
2. The speed of sound depends on the properties of the medium (e.g., density, elasticity).
3. The wavelength of the sound wave can be calculated using the formula: wavelength = speed of sound / frequency.
4. When the speed of sound is higher in the first medium and lower in the second medium, the wavelength will decrease according to the formula since the frequency is constant.

So, a transition from a medium with a higher speed of sound to a medium with a lower speed of sound would cause the wavelength of the sound wave to shorten.

To know more about properties of the medium:

https://brainly.com/question/23088538

#SPJ11

A sound wave transitioning from a medium with a higher speed of sound to a medium with a lower speed of sound will result in a shortening of the wavelength.

When a sound wave transitions from a medium with a higher speed of sound to a medium with a lower speed of sound, the wavelength of the sound wave will shorten.
Step-by-step explanation:
1. A sound wave is an oscillation of pressure that propagates through a medium.
2. The transition occurs when the sound wave moves from one medium to another.
3. The speed of sound in each medium depends on the medium's properties (density, elasticity, etc.).
4. If the sound wave moves from a medium with a higher speed of sound to a medium with a lower speed of sound, the wavelength will shorten.
5. This shortening occurs because the wave's frequency remains constant, and since the speed of sound has decreased, the wavelength must also decrease to maintain the relationship: speed = wavelength × frequency.

To learn more about the sound wave, refer:-

https://brainly.com/question/11797560

#SPJ11

(a) Electric room heaters use a concave mirror to reflect infrared (IR) radiation from hot coils. Note that IR follows the same law of reflection as visible light. Given that the mirror has a radius of curvature of 50.0 cm and produces an image of the coils 3.00 m away from the mirror, where are the coils?
(b) Find the magnification of the heater element in (b). Note that its large magnitude helps spread out the reflected energy.

Answers

(a) Coils are located 31.58 cm away from the mirror.

(b) Magnification is -9.50, indicating an inverted image, and the large magnitude helps spread out the reflected energy for effective heating.

(a) We can use the mirror equation to solve for the distance of the object (coils) from the mirror:

1/f = 1/do + 1/di

where f is the focal length (half the radius of curvature), do is the distance of the object from the mirror, and di is the distance of the image from the mirror.

Substituting the given values, we get:

1/25 = 1/do + 1/300

Solving for do, we get:

do = 31.58 cm

So the coils are 31.58 cm away from the mirror.

(b) The magnification, M, is given by:

M = -di/do

Substituting the given values, we get:

M = -3.00 m / 0.3158 m

M = -9.50

The negative sign indicates that the image is inverted. The large magnitude of the magnification means that the reflected energy is spread out over a large area, making the heater more effective at heating a room.

Learn more about Magnification

https://brainly.com/question/31595015

#SPJ4

a workman carries some lumber up a staircase. the workman moves 9.6 m vertically and 22 m horizontally. if the lumber weighs 45 n, how much work was done by the workman?

Answers

The total work done by the workman was 432 joules

The workman lifted the lumber a vertical distance of 9.6 m, which means he did work against gravity. The amount of work done is equal to the force (weight of the lumber) multiplied by the distance it was lifted:

Work = force x distance

Work = 45 N x 9.6 m

Work = 432 joules

In addition to lifting the lumber, the workman also moved it horizontally a distance of 22 m. However, since the lumber was not lifted or lowered during this movement, no work was done against gravity.

Therefore, this distance does not affect the amount of work done by the workman.

To learn more about : work

https://brainly.com/question/30121704

#SPJ11

hydrolysis is more common in a(n) _____ climate

Answers

Hydrolysis is a chemical reaction in which water is used to break down complex molecules into simpler ones.

This process is more common in a humid or wet climate. In such climates, water is readily available and tends to accumulate in soils and rocks, leading to the formation of aqueous solutions. These solutions can then react with various minerals and organic compounds, promoting hydrolysis. Moreover, the presence of high temperatures and abundant vegetation in tropical climates accelerates the process of hydrolysis.

This results in the decomposition of organic matter, which releases nutrients and minerals that can support plant growth. Overall, hydrolysis plays a crucial role in many environmental processes and is particularly important in regions with high moisture levels.

Learn more about complex molecules

https://brainly.com/question/30336127

#SPJ4

Water is utilised in a chemical procedure called hydrolysis to convert complicated molecules into simpler ones.

A humid or moist climate favours this procedure more frequently. In such environments, water is easily accessible and has a propensity to build up in rocks and soils, resulting in the creation of aqueous solutions. The subsequent reactions between these solutions and different minerals and organic molecules can encourage hydrolysis. Additionally, tropical areas' high temperatures and plenty of flora hasten the hydrolysis process.

This causes organic materials to decompose, releasing nutrients and minerals that can help plants flourish. Overall, hydrolysis is critical to many environmental processes and is especially significant in areas with high levels of moisture.

learn more about complicated molecules here:

https://brainly.com/question/13443071

#SPJ11

When a 0. 30 kg mass is suspended from a massless spring, the spring stretches a distance of 2. 0 cm. Let 2. 0 cm be the rest position for the mass-spring system. The mass is then pulled down an additional distance of 1. 5 cm and released. Calculate the total mechanical energy of the system in SI Units.

Spring constant can be found using Hooke's Law

Answers

The total mechanical energy of the system is 0.0066 J.

Using Hooke's Law, the spring constant can be calculated as k = F/x, where F is the weight of the mass and x is the displacement of the spring from its rest position.

In this case:

F = mg,

where m is the mass of the object and g is the acceleration due to gravity.

Therefore, k = (mg)/x.

Once the spring constant is known, the total mechanical energy of the system can be calculated as:

E = (1/2)kx^2.

Substituting the given values, we get

k = 14.7 N/m and x = 0.03 m.

Hence, the total mechanical energy of the system is

E = (1/2)kx^2 = 0.0066 J.

To know more about Hooke's Law, here

brainly.com/question/29126957

#SPJ4

which statement is true regarding the resolution of a grating? a. resolution increases with wavelength b. resolution decreases with number of grooves per mm c. resolution increases with number of grooves per mm d. resolution is not determined by the monochromator e. resolution increases with slit width

Answers

The correct statement regarding the resolution of a grating is that the resolution increases with the number of grooves per mm, the correct option is (c).

The resolution of a grating is defined as the ability to separate two closely spaced spectral lines or wavelengths. It is determined by the number of grooves per unit length on the grating surface, as well as the wavelength of the incident light and the angle of incidence.

A higher number of grooves per mm means that the grating will disperse the incoming light into more angles, resulting in higher resolution. Therefore, the number of grooves per mm is the primary factor that determines the resolution of a grating, the correct option is (c).

To learn more about resolution follow the link:

https://brainly.com/question/30753488

#SPJ4

The complete question is:

Which statement is true regarding the resolution of a grating?

a. resolution increases with wavelength

b. resolution decreases with number of grooves per mm

c. resolution increases with number of grooves per mm

d. resolution is not determined by the monochromator

e. resolution increases with slit width

Other Questions
A business that pays for its workers to attend a technical college is increasing its:human capital.technical knowledge.organizational skills.physical capital.human capital all wheel nuts must be tightened to the correct torque and in the proper _____________ The expression (h) (b, + b) gives the area of a trapezoid, with b, and b, representing the two base lengths of a trapezoid and h representing the height. Find the area of a trapezoid with base lengths 4 in. and 6 in. and a height of 8 in. (Lesson 10.2) Juan can run 38 yards in 5 seconds. If he keeps the same pace, how many yards can he run in 30 seconds? Responses on the centered zipper, what is the only part of the work that is done on the outside of the garment? managers can reduce the need for organizational rules and regulations by hiring the right people, providing training, developing management role models, and creating blank systems. multiple choice question. reward rule-based control corporate governance currently, the compensation for lost or damage airlines luggage has been negotiated by international agreement and is determined by which currency standard? which of the following is true of the motor skills that emerge during early childhood? question 60 options: children's fine motor skills are the primary determinants of their performance in sports. hopping is generally mastered by children at the age of 2. going up stairs is easier than going down. the skills that emerge in early childhood are independent of the achievements of infancy and toddlerhood. collins manufacturing has the following information: common stock is 2.5 million shares with a current price of $42 per share; the beta of the stock is 1.5; the standard deviation of the stock is 10.5%. market: the us treasury bill is yielding 2.8% and the expected return on the market is 10.8%. the corporate tax rate is 38%. what is the firm's expected return on equity? which kind of chinese and japanese paintings often found inspiration from poetry or the beliefs of daoism, which held nature in high esteem? Human Capital theory suggests that everyones income reflects individual choices about investments in education and training.True False The price of a commercial paper offering a payoff of$10,000 is $9,460. If the annualized investment rate is 7.8%, whendoes the paper mature? an educator wants to determine how effective tutoring is in raising students' grades in her class, so she offers free tutoring for those who want it. then she compares final exam grades for the group that took advantage of the tutoring and the group that did not. suppose the group participating in the tutoring received higher grades on the exam. does that show that the tutoring worked? if not, explain why not. the study shows that the tutoring worked. A car rental company's standard charge includes an initial fee plus an additional fee for each mile driven. The standard charge (in dollars) is given by the function , where is the number of miles driven.The company also offers an option to insure the car against damage. The insurance charge (in dollars) is given by the function .Let be the total charge (in dollars) for a rental that includes insurance. Write an equation relating to . Simplify your answer as much as possible. a college student goes to the campus health office complaining of diarrhea, lower right abdominal pain, and weight loss. suspecting crohn disease, the nurse will assess for which complication associated with this diagnosis? What reason did the government give for forcing the Native Americans to relocate in the 1830s? How much money would an investment of 300 at rate of 12 percent compound monthly be after 4 years in database design, the ideal primary key is short, numeric, and nonchanging. (True or False) Suppose that the function h is defined as follows.if-2-1h(x)= 012Graph the function h.-3.5if-2.5if-1.5if -0.5if 0.5 x1.5+X Two years ago, Blue Ltd sold of$1000par value that had an original maturity of 15 years and a coupon rate of Today these bonds are selling for$1,120. Determine the yield-to-maturity.a.9.77%b.7.92%c.8.44%d.10.29%e.8.55%